Location via proxy:   [ UP ]  
[Report a bug]   [Manage cookies]                
Download as pdf or txt
Download as pdf or txt
You are on page 1of 14

COSC 3451: Signals and Systems

Solutions to Assignment # 2: Time Domain Analysis of LTI Systems


Chapters 2 and 3.

1. The following CT systems are described using their input-output relationships between the input signal x(t)
and output y(t) . Determine if the CT systems are: (i) linear; (ii) time-invariant; (iii) stable; and (iv) causal.
i) y (t ) = x (t − 2)
ii) y (t ) = tx (t + 10)

⎧⎪ 0 t<0
iii) y (t ) = ⎨
⎪⎩ x (t ) − x (t − 5) t≥0
to
iv) y (t ) = ∫ x(λ)dλ + 2 x(t )
−to

d4y d3y d2y dy d 2x


v) + 3 + 5 + 3 + y (t ) = + 2 x(t ) + 1
dt 4 dt 3 dt 2 dt dt 2
Solution: i) y (t ) = x (t − 2)
Linearity:
The input-output relationships for two different inputs x1(t) and x2(t) are given by
Given x1 (t ) → y1 (t ) = x1 (t − 2)
and x 2 (t ) → y1 (t ) = x 2 (t − 2)
then αx1 (t ) + β x 2 (t ) → y 3 (t ) = [αx1 (t − 2) + β x 2 (t − 2)] = αy1 (t ) + β y 2 (t )
hence, system (i) is linear.
Time-invariance:
The output y1 (t ) for the input x1 (t ) = x(t − t 0 ) is given by
x1 (t ) = x(t − t 0 ) → y1 (t ) = x(t − t 0 − 2) = y (t − t 0 )
hence, system (i) is time-invariant.
Stability:
Assuming x(t ) < B x < ∞ , for (−∞ < t < ∞), then y (t ) =| x(t − 2) |< B x < ∞ . System (i) is stable.

Causality:
Calculating the output y(t) at t = t0 gives
y (t ) t =t = x(t 0 − 2)
0

implying that the the output y(t) at t = t0 only on the input ( t 0 − 2 ) in the past. System (i) is causal.

ii) y (t ) = tx(t + 10)


Linearity:
The input-output relationships for two different inputs x1(t) and x2(t) are given by
Given x1 (t ) → y1 (t ) = tx1 (t − 2)
and x 2 (t ) → y1 (t ) = tx 2 (t − 2)
then αx1 (t ) + βx 2 (t ) → y 3 (t ) = t [αx1 (t ) + βx 2 (t )] = αy1 (t ) + β y 2 (t )
hence, system (ii) is linear.
Time-invariance:
The output y1 (t ) for the input x1 (t ) = x(t − t 0 ) is given by

x1 (t ) = x(t − t 0 ) → y1 (t ) = tx1 (t ) = tx(t − t 0 + 10) ≠ y (t − t 0 )


hence, system (ii) is NOT time-invariant.
Stability:
Assuming x(t ) < B x < ∞ , for (−∞ < t < ∞), then y (t ) =| t || x(t + 10) |<| t | B x . System (ii) is NOT stable since the
value of the output approaches infinity as t → ∞ .
Causality:
Calculating the output y(t) at t = t0 gives
y (t ) t =t = t 0 x(t 0 + 10)
0

implying that the the output y(t) at t = t0 only on the input at ( t 0 + 10 ) in the future. System (ii) is NOT causal.

⎧ 0 t<0
iii) y (t ) = ⎨
⎩ x(t ) − x(t − 5) t ≥ 0
Linearity:
The input-output relationships for two different inputs x1(t) and x2(t) are given by
⎧ 0 t<0
Given x1 (t ) → y1 (t ) = ⎨
⎩ x1 (t ) − x1 (t − 5) t≥0
⎧ 0 t<0
and x 2 (t ) → y 2 (t ) = ⎨
⎩ x 2 (t ) − x 2 (t − 5) t≥0
⎧ 0 t<0
then αx1 (t ) + βx 2 (t ) → y 3 (t ) = ⎨ = αy1 (t ) + β y 2 (t )
⎩αx1 (t ) + βx 2 (t ) − αx1 (t − 5) + β x 2 (t − 5) t ≥ 0
hence, system (iii) is linear.
Time-invariance:
The output y1 (t ) for the input x1 (t ) = x(t − t 0 ) is given by

⎧ 0 t<0
x1 (t ) = x(t − t 0 ) → y1 (t ) = ⎨ = y (t − t 0 )
⎩ x(t − t 0 ) − x(t − t 0 − 5) t ≥ 0
hence, system (iii) is time-invariant.
Stability:
Assuming x(t ) < B x < ∞ , for (−∞ < t < ∞), then y (t ) = 2 B x < ∞. System (iii) is stable.

Causality:
Calculating the output y(t) at t = t0 gives
⎧ 0 t<0
y (t ) t =t == ⎨
0
⎩ x(t 0 ) − x(t 0 − 5) t ≥ 0
implying that the output y(t) at t = t0 only on the value of the input at t = t0 and t = t0 – 5. System (iii) is causal.
to

iv) y (t ) = ∫ x(λ)dλ + 2 x(t ) .


−to

Linearity:
The input-output relationships for two different inputs x1(t) and x2(t) are given by
to

Given x1 (t ) → y1 (t ) = ∫ x (λ)dλ + 2 x (t )
−to
1 1

to

and x 2 (t ) → y 2 (t ) = ∫x
−t o
2 (λ ) dλ + 2 x 2 (t )

to

then ∫
αx1 (t ) + β x 2 (t ) → y 3 (t ) == [αx1 (λ ) + β x 2 (λ )]dλ + 2[αx1 (t ) + βx 2 (t )] = αy1 (t ) + βy 2 (t )
−to

hence, system (iv) is linear.


Time-invariance:
The output y1 (t ) for the input x1 (t ) = x(t − t1 ) is given by
to to −t1

x1 (t ) = x(t − t1 ) → y1 (t ) = ∫ x(λ − t )dλ + 2 x(t − t ) = ∫ x(λ)dλ + 2 x(t − t ) ≠ y(t − t )


−to
1 1
−to −t1
1 1

hence, system (iv) is NOT time-invariant.


Stability:
to

Assuming x(t ) < B x < ∞ , for (−∞ < t < ∞), then y (t ) = ∫ x ( λ ) dλ + 2 B
−to
x < ∞. System (iv) is stable.

Causality:
Calculating the output y(t) at t = t0 gives
to

y (t ) t = t =
1 ∫ x(λ)dλ + 2 x(t )
−t o
1

implying that the output y(t) at t = t1 requires the values of the input at t = t1 and −t0 ≤ t ≤ t0, where t0 can possibly
be greater than t0. System (iv) is NOT causal.
2. The following DT systems are described using their input-output relationships between the input signal x[k]
and output y[k]. Determine if the DT systems are: (i) linear; (ii) time-invariant; (iii) stable; and (iv) causal.
i) y[k ] = 0.5 x[−2 + 6k ] + 0.5 x[2 + 6k ]
k +2
ii) y[k ] = ∑ x[m] − 2 | x[k ] |
m=k −2
iii) y[k ] = 2 x[ k ]
iv) y[k ] + 5 y[k − 1] + 9 y[k − 2] + 5 y[ k − 3] + y[k − 4] = 2 x[ k ] + 4 x[k − 1] + 2 x[k − 2]
Solution:
i) y[k ] = 0.5 x[−2 + 6k ] + 0.5 x[2 + 6k ]
Linearity:
The input-output relationships for two different inputs x1(t) and x2(t) are given by
Given x1 [ k ] → y 1 [ k ] = 0 . 5 x 1 [ − 2 + 6 k ] + 0 . 5 x 1 [ 2 + 6 k ]
and x 2 [ k ] → y 2 [ k ] = 0 . 5 x 2 [ − 2 + 6 k ] + 0 .5 x 2 [ 2 + 6 k ]
then α x1 [ k ] + β x 2 [ k ] → y 3 [ k ] = 0 .5(α x1 [ − 2 + 6 k ] + βα x 2 [ − 2 + 6 k ]) + 0 .5(α x1 [ 2 + 6 k ] + βα x 2 [ 2 + 6 k ]) = α y1 [ k ] + β y 2 [ k ]
hence, system (i) is linear.
Time-invariance:
The output y1[k] for the input x1[k] = x[k – k0] is given by
x1 [k ] → y1 [k ] = 0.5 x1 [ −2 + 6( k − k 0 )] + 0.5 x1 [2 + 6(k − k 0 )] = y[ k − k 0 ]
hence, system (i) is time-invariant.
Stability:
Assuming x[k ] < B x < ∞ , for (−∞ < t < ∞), then y[k ] = B x < ∞. System (i) is stable.

Causality:
The output at time k = k0 also depends on the input at k = 2 + 6k0 and k = − 2 + 6k0 . System (i) is NOT causal.
k +2
ii) y[k ] = ∑ x[m] − 2 | x[k ] |
m=k − 2

Linearity:
The input-output relationships for two different inputs x1(t) and x2(t) are given by
k +2
Given x1 [k ] → y1 [k ] = ∑ x [ m] − 2 | x [ k ] |
m=k −2
1 1

k +2
and x 2 [k ] → y 2 [k ] = ∑ x [ m] − 2 | x [ k ] |
m=k −2
2 2

k +2
then αx1 [k ] + βx 2 [k ] → y 3 [k ] = ∑ [αx [k ] + βx [k ]] − 2 | αx [k ] + βx [k ] |≠ αy [k ] + βy [k ]
m=k −2
1 2 1 2 1 2

hence, system (ii) is NOT linear. (Hint: |(−1)x[k]| = (-1)|x[k]|).


Time-invariance:
The output y1[k] for the input x1[k] = x[k – k0] is given by
k +2 k − k0 + 2
x1[k ] → y1[k ] = ∑
m=k −2
x[m − k 0 ] − 2 | x[ k − k 0 ] | = ∑ x[ p] − 2 | x[k − k
p = k − k0 − 2
0 ] |= y[k − k 0 ]

hence, system (ii) is time-invariant.


Stability:
k +2
Assuming x[k ] < B x < ∞ , for (−∞ < t < ∞), then y[k ] < ∑ x[m − k
m=k −2
0] + 2 | x[ k − k 0 ] |< 7 B x < ∞. System (ii) is

stable.
Causality:
The output at time k = k0 also depends on the input at k = k0 + 2 and k = k = k0 − 2. System (ii) is NOT causal.

(iii) y[k ] = 2 x[ k ]
Linearity:
The input-output relationships for two different inputs x1(t) and x2(t) are given by
Given x1 [k ] → y1 [k ] = 2 x1[ k ]
and x 2 [ k ] → y 2 [ k ] = 2 x2 [ k ]
then αx1 [k ] + β x 2 [k ] → y 3 [k ] = 2 αx1[ k ]+βx2 [ k ] = 2 α 2 x1 [ k ] + 2 β 2 x2 [ k ] ≠ αy1 [k ] + βy 2 [k ]
hence, system (iii) is NOT linear.
Time-invariance:
The output y1[k] for the input x1[k] = x[k – k0] is given by
x1 [k ] → y1 [k ] = 2 x[ k − k0 ] = y[k − k 0 ]
hence, system (iii) is time-invariant.
Stability:

Assuming x[k ] < B x < ∞ , for (−∞ < t < ∞), then y[k ] < 2 x[ k ] < B y < ∞. System (iii) is stable.

Causality:
The output at time k = k0 also depends on the input at k = k0. System (iii) is causal. In fact, it is memoryless.

(iv) Constant coefficients difference equations represent a linear and time-invariant system. Such a system is also
causal if initial conditions are zero.

3. For a CT linear, time invariant (LTI) system, an input x(t) produces output y(t) shown in Fig. P2.6. Sketch
the outputs for the following set of inputs.
i) 5 x(t )
ii) 0.5 x(t − 1) + 0.5 x(t + 1)
iii) x(t + 1) − x(t − 1)
dx(t )
iv) + 3 x(t )
dt
y(t)
1

t
−1 1
Figure P3: CT output y(t) for problem 3.
Solution: Based on the linearity and time invariance property, the outputs are given by
i) 5 x(t ) → y1 (t ) = 5 y (t ). .

ii) 0.5 x(t − 1) + 0.5 x(t + 1) → y 2 (t ) = 0.5 y (t − 1) + 0.5 y (t + 1).


iii) x(t + 1) − x(t − 1) → y 3 (t ) = y (t + 1) − y (t − 1).

dx(t ) dy (t )
iv) + 3x(t ) → y 3 (t ) = + 3 y (t ).
dt dt
The resulting waveforms are plotted below.

y1(t) = 5y (t)
5

t
−5 −4 −3 −2 −1 0 1 2 3 4 5 6

(i)

y1(t) = 0.5y (t − 1) + 0.5y (t + 1)

0.5
t
−5 −4 −3 −2 −1 0 1 2 3 4 5 6

(ii)

y1(t) = y (t − 1) − y (t + 1)

t
−5 −4 −3 −2 −1 0 1 2 3 4 5 6
−1

(iii)

y1(t) = dy/dt + 3x(t)


3

t
−5 −4 −3 −2 −1 0 1 2 3 4 5 6

(iv)

4. For a DT linear, time invariant (LTI) system, an input x[k] produces output y[k] shown in Fig. P2.7. Sketch
the outputs for the following set of inputs.
i) 4 x[k − 1]
ii) 0.5 x[k − 2] + 0.5 x[k + 2]
iii) x[k + 1] − 2 x[k ] + x[k − 1]
iv) x[− k ]
y[k]
4
2
−1
k
−2 1 2
−2
Figure P4: DT output y[k] for problem 4.
Solution: Based on the linearity and time invariance property, the outputs are given by
i) 4 x[ k − 1] → y1 [k ] = 4 y[k − 1]. .

ii) 0.5 x[k − 2] + 0.5 x[k − 2] → y 2 [ k ] = 0.5 y[k − 2] + 0.5 y[k + 2].

iii) x[k + 1] − 2 x[k ] + x[k − 1] → y 3 [k ] = y[k + 1] − 2 y[ k ] + y[k − 1].


iv) x[−k ] → y 4 [k ] = y[−k ].
The resulting waveforms may be plotted from the above results.

5. Determine if the following CT systems are invertible. If yes, find the inverse systems.
i) y (t ) = 3 x(t + 2)
t
ii) y (t ) = ∫ x( τ − t ) dt
0
iii) y (t ) = x(t )
iv) y (t ) = cos(2πx(t ))
Solution:
(i) Invertible with the inverse system given by x (t ) = 1
3 y (t − 2) .
t
(ii) y (t ) = ∫ x( τ − t ) dt is not invertible.
0

(iii) y (t ) = x (t ) = a 2 (t ) + b 2 (t ) assuming x (t ) = a (t ) + jb(t ) . Note that y (t ) provides the magnitude of x (t ) .


Unless, we know the phase, x (t ) cannot be determined uniquely. Therefore, the system is NOT invertible.

(iv) y (t ) = cos(2πx(t )) is not invertible since for inputs x(t) = −π/2 and x(t) = π/2, the output is the same.

6. Determine if the following DT systems are invertible. If yes, find the inverse systems.
i) y[k ] = ( k + 1) x[k + 2]
k
ii) y[k ] = ∑ x[m + 2]
m=0
iii) y[k ] = x[k + 2] + 2 x[k + 1] − 6 x[k ] + 2 x[k − 1] + x[k − 2]
iv) y[k ] + 2 y[k − 1] + y[k − 1] = x[k ]
Solution:
(i) Not invertible since y[−1] = 0 irrespective of the value of x[−1].
(ii) Not invertible since the outputs will be the same for two different inputs say x[k] = u[k] and x[k] = u[k +
2].
(iii) Invertible.
(iv) Invertible
Inverse systems for parts (iii) and (iv) will be determined once the Fourier transforms are covered.

7. For each of the following differential equations modeling an LTIC system, determine the zero-input
response, zero-output response and overall response of the systems for the specified input x(t) and initial
conditions. What is the steady state response of the LTIC system?
i) y (t ) + 4 y& (t ) + 8 y (t ) = x& (t ) + x (t ) with x (t ) = e −4 t u(t ), y (0) = 0, and y& (0) = 0.
&&
ii) y (t ) + 6 y& (t ) + 4 y (t ) = x& (t ) + x (t ) with x (t ) = cos(6t )u(t ), y (0) = 2, and y& (0) = 0.
&&
iii) y (t ) + 4 y (t ) = 5 x (t ) with x (t ) = 4te− t u(t ), y (0) = −2, and y& (0) = 0.
&&
iv) y (t ) + 2 &&y (t ) + y (t ) = x (t ) with x (t ) = 2u(t ), y (0) = 0, and y& (0) = 1.
&&&&

Solution:
We solve part (i) here. Remaining parts are similar to part (i) but the students are encouraged to practice.
i) Zero-input response of the system: The characteristic equation of system (i) is

s 2 + 4s + 8 = 0
which has roots at s = −2 ± j2. −3. The zero-input response is therefore given by
y zi (t ) = ( Ae ( −2 + j 2 )t + Be ( −2− j 2 )t )u (t )
where A and B are constants. To calculate the value of the constants, we substitute the initial conditions y(0) = 0
and y•(0) = 0 in the above equation. The resulting simultaneous equations are
A+ B=0
(−2 + j 2) A + (−2 − j 2) B = 0
that has a solution, A = 0 and B = 0. The zero-input response is therefore given by
y zi (t ) = 0.
Zero-state response of the system: To calculate the zero-state response of the system, the initial conditions are
assumed to be zero. Hence
d2y dy
2
+4 + 8 y (t ) = x • (t ) + x(t ) (q1)
dt dt
with initial conditions, y(0) = 0 and and y•(0) = 0. The combined input is given by
⎧− 3e −4t t>0
x • (t ) + x(t ) = ⎨
⎩ 0 t < 0.
The homogenous solution of Eq. (q1) is given by
y zsh ( t ) = ( C 1 e ( − 2 + j 2 ) t + C 2 e ( − 2 − j 2 ) t ) u ( t ) .

where C1 and C2 are constants. The particular solution for input x • (t ) + x(t ) = −3e−4tu(t) is of the form
y zs( p ) (t) = Ke −4t u (t ) . Substituting the particular solution in Eq. (q1) provides the value of K as 3/8. The zero-
state response of the system is therefore given by
y zs (t ) = y zsh (t ) + y zsp (t ) = (C1e ( −2+ j 2)t + C 2 e ( −2− j 2)t + 83 e −4t )u (t ) .

To compute the values of constants C1 and C2, we use the initial conditions, y(0−) = 0 and y•(0) = 0. Substituting
the initial conditions in yzs(t) leads to the following simultaneous equations
C1 + C 2 + 83 = 0
(−2 + j 2)C1 + (−2 − j 2)C 2 − 32 = 0

with solution: C1 = (0.1875 + j0.1875) and C2 = (0.1875 + j0.1875).

Overall response of the system: The overall response of the system can be obtained by summing up the zero-
input and zero-state responses, and is given by
y zs (t ) == (C1e ( −2+ j 2)t + C 2 e ( −2− j 2)t + 83 e −4t )u (t )

with C1 = (0.1875 + j0.1875) and C2 = (0.1875 + j0.1875).

8. Determine the output y(t) for the following pairs of input signals x(t) and impulse responses h(t).
i) x(t ) = u (t ) h(t ) = u (t )
ii) x(t ) = u (t ) − 2u (t − 1) + u (t − 2) h(t ) = u (t + 1) − u (t − 1)
iii) x(t ) = e 2t u (−t ) h(t ) = e −3t u (t )
iv) x(t ) = sin(2πt )(u (t − 2) − u (t − 5) ) h(t ) = u (t ) − u (t − 2)

Solution:
i) The output y(t) is given by
∞ ∞
y ( t ) = u( t ) * u( t ) = ∫
−∞
u(τ )u(t − τ )dτ = ∫ u(t − τ )dτ
0

⎧ 0 t<0
⎪t
=⎨ [Q u(t − τ ) = 1 if τ ≤ t ]
⎪ ∫ 1dτ t > 0
⎩0
⎧⎪0 t < 0
=⎨ = tu(t ) = r (t )
⎪⎩ t t > 0
The aforementioned convolution can also be computed graphically.
ii) The output y(t) is given by
y (t ) = [u (t ) − 2u (t − 1) + u (t − 2)] ∗ [u (t + 1) − u (t − 1)]
Using the properties of the convolution integral, the output is expressed as
y (t ) = [u (t ) ∗ u (t + 1)] − [u (t ) ∗ u (t − 1)] − 2[u (t − 1) ∗ u (t + 1)] + 2[u (t − 1) ∗ u (t − 1)] + [u (t − 2) ∗ u (t + 1)] − [u (t − 2) ∗ u (t − 1)]
Based on the results of part (i), the overall output is given by
y (t ) = r (t + 1) − r (t − 1) − 2r (t ) + 2r (t − 2) + r (t − 1) − r (t − 3) .
iii) The output y(t) is given by
∞ 0

∫ e2τ u( −τ )e−3( t −τ ) u(t − τ )dτ = e−3t ∫e



y (t ) = e2t u( −t ) * e−3t u(t ) = u(t − τ )dτ
−∞ −∞

⎧ t 5τ
⎪ ∫ e dτ t<0
⎪⎧ 5 e t <0
0 1 5t
⎪ −∞
= e−3t ∫ e5τ u(t − τ )dτ = e−3t × ⎨ = e −3 t × ⎨
−∞
0
⎪ 5τ ⎩⎪ 5
1
t ≥0
⎪ ∫ e dτ t≥0
⎩ −∞
⎧⎪ 15 e2t t <0
=⎨ = 15 e2t u( −t ) + 15 e−3t u(t )
1 −3t
⎪⎩ 5 e t ≥0

iv) The output y(t) is given by

∞ ⎡ ⎤
y (t ) = x(t ) * h(t ) = (τ − 2) − u (τ − 5) ⎥ [u (t − τ ) − u (t − τ − 2) ] dτ
∫ sin(2πτ ) ⎢⎢u144 42444 3⎥
−∞
⎣ = 0 for τ < 2,τ > 5 ⎦
5 5 5
= ∫ sin(2πτ ) [u (t − τ ) − u (t − τ − 2) ] dτ = ∫ sin(2πτ )u (t − τ )dτ − ∫ sin(2πτ )u (t − τ − 2)dτ
2 2 2
144 424443 1444 424444 3
=A =B
Calculating Term A and Term B separately, we get


⎪0 t≤2
⎪t ⎧0 t≤2
⎪ ⎪ 1−cos 2π t
A = ⎨ ∫ sin(2πτ )dτ 2 ≤ t ≤ 5 = ⎨ 2π 2≤t ≤5
⎪2 ⎪
⎪5 ⎩ 0 t ≥5
⎪ sin(2πτ )dτ
⎪∫
t ≥5
⎩2


⎪ 0 t−2≤ 2
⎪t − 2 ⎧ 0 t≤4 ⎧0 t≤4
⎪ ⎪ 1−cos 2π (t − 2) ⎪
B = ⎨ ∫ sin(2πτ )dτ 2 ≤ t − 2 ≤ 5 = ⎨ 2π 4 ≤ t ≤ 7 = ⎨ 1−cos

2π t
4≤t ≤7
⎪2 ⎪ ⎪
⎪ 5 ⎩ 0 t≥7 ⎩ 0 t≥7
⎪ sin(2πτ )dτ
⎪∫
t −2≥5
⎩ 2
The overall output is therefore given by

⎧0 t ≤ 2, t ≥ 7
⎪1
⎪ 2π (1 − cos 2π t ) 2≤t ≤4
Therefore, y (t ) = A − B = ⎨
⎪0 4≤t ≤5
⎪ 1
⎩− 2π (1 − cos 2π t ) 5≤t ≤7
9. Determine whether the LTIC systems characterized by the following impulse responses are memoryless,
causal and stable. Justify your answer.
i) h1(t ) = δ (t ) + e −5t u(t )
ii) h3(t ) = e −5t sin(2π t )u(t )
−2 t
iii) h 4(t ) = e + u(t + 1) − u(t − 1)
iv) h5(t ) = t [ u(t + 4) − u(t − 4)]
v) h7(t ) = cos(5t )u(t )
h8(t ) = 0.95
t
vi)

Solutions:
i) System h1(t) is NOT memoryless since h1(t) ≠ 0 for t ≠ 0.
System h1(t) is causal since h1(t) = 0 for t < 0.
∞ ∞ ∞
6
System h1(t) is stable since ∫
−∞
| h1(t ) | dt = ∫
−∞

δ(t )dt + e −5t u (t )dt =
−∞
5
<∞

ii) System h3(t) is NOT memoryless since h3(t) ≠ 0 for t ≠ 0.


System h3(t) is causal since h3(t) = 0 for t < 0.
∞ ∞ ∞
System h3(t) is stable since ∫
−∞
| h3(t ) | dt = ∫
−∞

e −5t sin( 2πt )u (t )dt + e −5t sin( 2πt )dt < ∞
0

iii) System h4(t) is NOT memoryless since h4(t) ≠ 0 for t ≠ 0.


System h4(t) is NOT causal since h4(t) ≠ 0 for t < 0.

System h4(t) is stable since ∫ | h4(t ) | dt =< ∞
−∞

iv) System h5(t) is NOT memoryless since h5(t) ≠ 0 for t ≠ 0.


System h5(t) is NOT causal since h5(t) ≠ 0 for t < 0.
∞ 4
System h3(t) is stable since ∫ | h5(t ) | dt = ∫ tdt = 16 < ∞
−∞ −4

v) System h7(t) is NOT memoryless since h7(t) ≠ 0 for t ≠ 0.


System h7(t) is causal since h7(t) = 0 for t < 0.
∞ ∞
System h3(t) is NOT stable since ∫ | h7(t ) | dt = ∫ | cos(5t )dt is infinite.
−∞ 0

vi) System h8(t) is NOT memoryless since h8(t) ≠ 0 for t ≠ 0.


System h8(t) is NOT causal since h8(t) ≠ 0 for t < 0.
∞ ∞ 0

∫ | h7(t ) | dt = ∫ 0.95 dt + ∫ 0.95


−t
System h8(t) is NOT stable since t
dt < ∞.
−∞ 0 −∞

10. For the four CT signals shown in Fig. P10, determine the following convolution.
i) y1 (t ) = v(t ) ∗ v(t )
ii) y2 (t ) = w(t ) ∗ w(t )
iii) y3 (t ) = x(t ) ∗ z (t )
iv) y4 (t ) = w(t ) ∗ z (t )
v) y6 (t ) = v(t ) ∗ z (t )

v(t) w(t)
1 1
(1 + t) (1 − t)
e2t e−2t

t t
−1 0 1 −1 0 1
(a) (b)
x(t) z(t)
1 1

−1
t t
0 1 2 0 1

−1 −1

(c) (d)

Fig. P10: CT signals for Problem 10.

Solution:
We solve part (i) in the solution. The remaining can be solved by following the steps shown here.

Step 1: v(τ)
1
e2τ e−2τ

τ
−1 0 1

Step 2: v(−τ)
1
e2τ e−2τ

τ
−1 0 1
Step 3: v(t −τ)
1
e2(t−τ) e−2(t−τ)

τ
t −1 t t +1

Step 5a: Case 1: (t + 1) < −1


y(t)
1
e2τ e−2τ

τ
t −1 t t +1 −1 0 1

Step 5b: Case 2: (t + 1) > −1


(t + 1) < 0 y(t)
1
e2τ e−2τ

τ
t −1 t −1 t +1 0 1

Step 5c: Case 3: (t + 1) > 0


(t + 1) < 1 y(t)
1
e2τ e−2τ

τ
t −1 −1 t 0 t +1 1

Step 5d: Case 4: (t + 1) > 1


(t – 1) < 0 y(t)
1
e2τ e−2τ

τ
−1 t −1 0 1 t +1

Step 5e: Case 5: (t − 1) > 0


(t – 1) < 1 y(t)
1
e2τ e−2τ

τ
−1 0 t −1 1 t +1
Step 5f: Case 6: (t − 1) > 1
y(t)
1
e2τ e−2τ

τ
−1 0 1t −1 t +1

From Step 5a: For (t < −2), y(t) = 0


From Step 5b: For (−2 < t < −1),
t +1 t +1 t +1
⎡ e 4τ ⎤ − 2t ⎡ e − e − 4 ⎤ 1 2t + 4
[ ]
4 ( t +1)
y (t ) = ∫
−1
e 2 τ e − 2 ( t − τ ) dτ = e − 2 t ∫
−1
e 4 τ dτ = e − 2 t ⎢ ⎥
⎣ 4 ⎦ −1
= e ⎢
⎣ 4
⎥= e
⎦ 4
− e − ( 2t − 4 ) .

From Step 5c: For (−1 < t < −0),


t 0 t +1

∫ ∫
y (t ) = e 2 τ e 2(t − τ) dτ + e 2 τ e − 2 (t − τ) dτ + ∫e
− 2 τ − 2 (t − τ )
e dτ .
−1 t 0

From Step 5d: For (0 < t < 1),


0 t 1

∫e ∫ ∫
2 τ 2(t − τ) − 2 τ 2(t − τ)
y (t ) = e dτ + e e dτ + e − 2 τ e − 2 (t − τ) dτ .
t −1 0 t

From Step 5e: For (1 < t < 2),


1

∫e
− 2 τ 2(t − τ)
y (t ) = e dτ
t −1

From Step 5f: For ( t > 2), y(t) = 0.


For each of the above cases, calculate y(t) and sketch it as a function of t.

You might also like